Self Studies

Physics Test - 10

Result Self Studies

Physics Test - 10
  • Score

    -

    out of -
  • Rank

    -

    out of -
TIME Taken - -
Self Studies

SHARING IS CARING

If our Website helped you a little, then kindly spread our voice using Social Networks. Spread our word to your readers, friends, teachers, students & all those close ones who deserve to know what you know now.

Self Studies Self Studies
Weekly Quiz Competition
  • Question 1
    4 / -1
    The work function of a substance is \(4.0{eV}\). The longest wavelength of light that can cause photoelectron emission from this substance is approximately:
    Solution

    Given,

    The work function of a substance,

    \(=4.0{eV}\)

    The minimum energy of incident radiations, required to eject the electrons from the metallic surface is defined as the work function of that surface.

    \(W_{0}=h v_{0}=\frac{h c}{\lambda_{0}}\)

    where \(\lambda_{0}=\) threshold wavelength.

    And\(W_{0}(e V)\) =Work function in electron volt,

    \(\lambda_{0}=\frac{\mathrm{hc}}{\mathrm{W}}\)\(=\frac{\left(6.62 \times 10^{-34}\right)\left(3 \times 10^{8}\right)}{4 \times 1.6 \times 10^{-19}}\)

    \(=3.10 \times 10^{-7} \mathrm{~m}=310 \times 10^{-9} \mathrm{~m}=310 \mathrm{~nm}\)

  • Question 2
    4 / -1

    Which of the following process is used to do maximum work done on the ideal gas if the gas is compressed to half of its initial volume?

    Solution

    The \(P-V\) diagram of adiabatic process, isothermal process and isobaric process. 

    Work done in process= area enclosed by \(P-V\) diagram with volume axis.Since area under the curve is maximum for adiabatic process, so work done on the gas will be maximum for adiabatic process.

  • Question 3
    4 / -1

    A circular coil has moment of inertia \(0.8 {kg} {m}^{2}\) around any diameter and is carrying current to produce a magnetic moment of \(20 {Am}^{2}\). The coil is kept initially in a vertical position and it can rotate freely around a horizontal diameter. When a uniform magnetic field of \(4 \mathrm{T}\) is applied along the vertical, it starts rotating around its horizontal diameter. The angular speed the coil acquires after rotating by \(60^{\circ}\) will be:

    Solution

  • Question 4
    4 / -1

    Point of maximum positive displacement in a transverse wave is called:

    Solution

    In a transverse waves particle of the medium vibrate up and down in the vertical direction whereas it is propagating along the horizontal direction. So, in a transverse wave, a crest is a part where particle rises from its mean position and has the maximum positive displacement whereas trough is a part where particle dips below mean position and has the maximum negative displacement.

  • Question 5
    4 / -1

    Work is best defined as:

    Solution

    The correct sentence is that work has only magnitude but no direction.

    • Work is donewhen the body moves with the application of external force or the moving body stop after theexternal forceapplied.
    • Work done is thedot productof force and displacement.
    • W = F ×d
      1. Where F = force applied
      2. d = displacement
    • The dot product of vector quantities is always scalarwhich means work is having only magnitude but no direction.
  • Question 6
    4 / -1

    The energy stored in a 50 mH inductor carrying a current of 4 A will be:

    Solution

    Given,

    \(\mathrm{L}=50 \mathrm{mH}\)

    \(=50 \times 10^{-3} \mathrm{H}\)

    \(\mathrm{I}=4 \mathrm{~A}\)

    Now,

    We know that,

    Then,

    \(U=\frac{1}{2}LI^{2}\)

    The magnetic potential energy of inductor \((U)\) is:

    \(U=\frac{1}{2} \times 50 \times 10^{-3} \times(4)^{2}\)

    \(\Rightarrow U=\frac{1}{2} \times 50 \times 10^{-3} \times 16\)

    \(\Rightarrow U=400 \times 10^{-3} \mathrm{H}\)

    \(\Rightarrow U=0.4 \mathrm{~J}\)

  • Question 7
    4 / -1

    A proton and an electron both are moving upward and enters into a uniform magnetic field. The direction of the magnetic field is perpendicular to the plane of paper and inward. Then both the particles will move:

    Solution

    Since the proton and electron, both are moving upward and the direction of the magnetic field is perpendicular to the plane of paper and inward, so we can say that both the particles are moving perpendicular to the magnetic field.

    When the velocity of a charged particle is perpendicular to a magnetic field, it describes a circle.

    Therefore we can say that both the particles will move in a circular path.

    The direction of the magnetic force on the moving charged particle in the magnetic field depends on the direction of velocity and the direction of the magnetic field but it is independent of the nature of the charge. 

    By the right-hand thumb rule, we can find out that the direction of the magnetic force. So the direction of the magnetic force on the proton and the electron will be such that it tends to rotate both the particles in the anticlockwise direction when viewed in the direction of the magnetic field.

  • Question 8
    4 / -1

    When light enters from rarer medium to denser medium then its:

    Solution

    We know the relation between frequency \((f)\) wavelength \((\lambda)\) and speed \((v)\):

    \(v=f \lambda\)

    When light passes from rarer medium to denser medium frequency remain the same but speed decrease \(v=\) constant

    \(\therefore \lambda\) decreases

    • Option (B) is correct because in refraction frequency is the same and the wavelength decreases as light travel from rarer to denser.
    • Option (A) is incorrect because frequency doesn't change in refraction.
    • Option (C) is incorrect because wavelength decreases as light travel from rarer to denser.
    • Option (D) is incorrect because frequency doesn't change in refraction.
  • Question 9
    4 / -1

    A body of mass \(M\) is kept on a rough horizontal surface (friction coefficient \(=\mu\) ). A person is trying to pull the body by applying a horizontal force but the body is not moving. The force by the surface on the body is \(F\), where:

    Solution

    Free body diagram is given as,

      

    If \(F_{1}=0\)

    So contact force will only have Normal which is equal to \(\mathrm{Mg}\).

    So \(F\) will be \(\mathrm{Mg}\).

    Now as \(\mathrm{F}_{1}\) increases \(f_r\) keeps increasing till it reaches limiting value i.e., \(\mu \mathrm{N}=\mu \mathrm{mg}\).

    So, in that case, contact force \(F\) will be,

    \(\sqrt{N^{2}+f_{r}^{2}}=\sqrt{(M g)^{2}+(\mu M g)^{2}}\)

    \(F=M g \sqrt{1+\mu^{2}}\)

    This means contact force \(F\) will lie between \({Mg} \leq  {F} \leq {Mg} \sqrt{1+\mu^{2}}\).

  • Question 10
    4 / -1

    In an n–type semiconductor, the electron concentration (no) = 1015 cm–3 and the intrinsic carrier concentration (ni) is 1010 cm–3. The mean life time of minority carriers is 10–6 sec. The steady state excess hole concentration due to constant light illumination is dp = 4 × 105 cm–3. The steady state electron – hole recombination rate is __________.

    Solution

    Recombination rate \((\mathrm{R})=\frac{P_{n}}{\tau_{P}}\)

    \(\begin{aligned}&P_{n}=P_{n o}+d P_{n} \\&=\frac{10^{20}}{10^{15}}+\left(4 \times 10^{5}\right) \mathrm{cm}^{-3}\end{aligned}\)

    \(=5 \times 10^{5} \mathrm{~cm}^{-3}\)

    \(\mathrm{R}=\frac{P_{n}}{z_{P}}=\frac{5 \times 10^{5}}{10^{-6}}=5 \times 10^{11} \mathrm{~cm}^{-3} \mathrm{~s}^{-1}\)

Self Studies
User
Question Analysis
  • Correct -

  • Wrong -

  • Skipped -

My Perfomance
  • Score

    -

    out of -
  • Rank

    -

    out of -
Re-Attempt Weekly Quiz Competition
Self Studies Get latest Exam Updates
& Study Material Alerts!
No, Thanks
Self Studies
Click on Allow to receive notifications
Allow Notification
Self Studies
Self Studies Self Studies
To enable notifications follow this 2 steps:
  • First Click on Secure Icon Self Studies
  • Second click on the toggle icon
Allow Notification
Get latest Exam Updates & FREE Study Material Alerts!
Self Studies ×
Open Now